Continuity of partial derivatives at (0,0)












2












$begingroup$


For the function $$f = begin {cases} frac{xy}{x^2+y^2}, text{if } (x,y) neq (0,0) \ 0, text{if } (x,y) = (0,0) end {cases}$$
show that $frac{partial f}{partial x}$ and $frac{partial f}{partial y}$ are not continuous at (0,0).



My attempt:
I found $$frac{partial f}{partial x}(x,y)=frac{y^3-x^2y}{(x^2+y^2)^2}$$
At $(x,y)=(0,0)$ $$frac{partial f}{partial x}(0,0)=lim_{trightarrow 0}frac{f(0+t,0)-f(0,0)}{t}=0.$$
Similarly $frac{partial f}{partial y}(0,0)=0$



Then $$lim_{(x,y)rightarrow(0,0)}frac{partial f}{partial x}(x,y)=lim_{(x,y)rightarrow(0,0)}frac{y^3-x^2y}{(x^2+y^2)^2}= text{ (consider restriction to the line } x = my) = lim_{yrightarrow 0}frac{y^3-m^2y^3}{(m^2y^2+y^2)^2}=lim_{yrightarrow 0}frac{y^3(1-m^2)}{(m^2+1)^2y^4}=infty neq frac{partial f}{partial x}(0,0).$$
Thus $frac{partial f}{partial x}$ is not continuous at (0,0). Similarly, for $frac{partial f}{partial y}.$



Is it correct? Also is it sufficient to show the inequality of the limit and the value of the partial, considering only one restriction to the line (in this case $x=my$)? Or should we consider other restrictions as well?










share|cite|improve this question









$endgroup$












  • $begingroup$
    Since the limit should be the same irrespective of the approach chosen, it is enough to consider limit along $x=my$ here.
    $endgroup$
    – Exp ikx
    Feb 1 at 19:20


















2












$begingroup$


For the function $$f = begin {cases} frac{xy}{x^2+y^2}, text{if } (x,y) neq (0,0) \ 0, text{if } (x,y) = (0,0) end {cases}$$
show that $frac{partial f}{partial x}$ and $frac{partial f}{partial y}$ are not continuous at (0,0).



My attempt:
I found $$frac{partial f}{partial x}(x,y)=frac{y^3-x^2y}{(x^2+y^2)^2}$$
At $(x,y)=(0,0)$ $$frac{partial f}{partial x}(0,0)=lim_{trightarrow 0}frac{f(0+t,0)-f(0,0)}{t}=0.$$
Similarly $frac{partial f}{partial y}(0,0)=0$



Then $$lim_{(x,y)rightarrow(0,0)}frac{partial f}{partial x}(x,y)=lim_{(x,y)rightarrow(0,0)}frac{y^3-x^2y}{(x^2+y^2)^2}= text{ (consider restriction to the line } x = my) = lim_{yrightarrow 0}frac{y^3-m^2y^3}{(m^2y^2+y^2)^2}=lim_{yrightarrow 0}frac{y^3(1-m^2)}{(m^2+1)^2y^4}=infty neq frac{partial f}{partial x}(0,0).$$
Thus $frac{partial f}{partial x}$ is not continuous at (0,0). Similarly, for $frac{partial f}{partial y}.$



Is it correct? Also is it sufficient to show the inequality of the limit and the value of the partial, considering only one restriction to the line (in this case $x=my$)? Or should we consider other restrictions as well?










share|cite|improve this question









$endgroup$












  • $begingroup$
    Since the limit should be the same irrespective of the approach chosen, it is enough to consider limit along $x=my$ here.
    $endgroup$
    – Exp ikx
    Feb 1 at 19:20
















2












2








2





$begingroup$


For the function $$f = begin {cases} frac{xy}{x^2+y^2}, text{if } (x,y) neq (0,0) \ 0, text{if } (x,y) = (0,0) end {cases}$$
show that $frac{partial f}{partial x}$ and $frac{partial f}{partial y}$ are not continuous at (0,0).



My attempt:
I found $$frac{partial f}{partial x}(x,y)=frac{y^3-x^2y}{(x^2+y^2)^2}$$
At $(x,y)=(0,0)$ $$frac{partial f}{partial x}(0,0)=lim_{trightarrow 0}frac{f(0+t,0)-f(0,0)}{t}=0.$$
Similarly $frac{partial f}{partial y}(0,0)=0$



Then $$lim_{(x,y)rightarrow(0,0)}frac{partial f}{partial x}(x,y)=lim_{(x,y)rightarrow(0,0)}frac{y^3-x^2y}{(x^2+y^2)^2}= text{ (consider restriction to the line } x = my) = lim_{yrightarrow 0}frac{y^3-m^2y^3}{(m^2y^2+y^2)^2}=lim_{yrightarrow 0}frac{y^3(1-m^2)}{(m^2+1)^2y^4}=infty neq frac{partial f}{partial x}(0,0).$$
Thus $frac{partial f}{partial x}$ is not continuous at (0,0). Similarly, for $frac{partial f}{partial y}.$



Is it correct? Also is it sufficient to show the inequality of the limit and the value of the partial, considering only one restriction to the line (in this case $x=my$)? Or should we consider other restrictions as well?










share|cite|improve this question









$endgroup$




For the function $$f = begin {cases} frac{xy}{x^2+y^2}, text{if } (x,y) neq (0,0) \ 0, text{if } (x,y) = (0,0) end {cases}$$
show that $frac{partial f}{partial x}$ and $frac{partial f}{partial y}$ are not continuous at (0,0).



My attempt:
I found $$frac{partial f}{partial x}(x,y)=frac{y^3-x^2y}{(x^2+y^2)^2}$$
At $(x,y)=(0,0)$ $$frac{partial f}{partial x}(0,0)=lim_{trightarrow 0}frac{f(0+t,0)-f(0,0)}{t}=0.$$
Similarly $frac{partial f}{partial y}(0,0)=0$



Then $$lim_{(x,y)rightarrow(0,0)}frac{partial f}{partial x}(x,y)=lim_{(x,y)rightarrow(0,0)}frac{y^3-x^2y}{(x^2+y^2)^2}= text{ (consider restriction to the line } x = my) = lim_{yrightarrow 0}frac{y^3-m^2y^3}{(m^2y^2+y^2)^2}=lim_{yrightarrow 0}frac{y^3(1-m^2)}{(m^2+1)^2y^4}=infty neq frac{partial f}{partial x}(0,0).$$
Thus $frac{partial f}{partial x}$ is not continuous at (0,0). Similarly, for $frac{partial f}{partial y}.$



Is it correct? Also is it sufficient to show the inequality of the limit and the value of the partial, considering only one restriction to the line (in this case $x=my$)? Or should we consider other restrictions as well?







real-analysis multivariable-calculus partial-derivative






share|cite|improve this question













share|cite|improve this question











share|cite|improve this question




share|cite|improve this question










asked Feb 1 at 19:13









dxdydzdxdydz

49110




49110












  • $begingroup$
    Since the limit should be the same irrespective of the approach chosen, it is enough to consider limit along $x=my$ here.
    $endgroup$
    – Exp ikx
    Feb 1 at 19:20




















  • $begingroup$
    Since the limit should be the same irrespective of the approach chosen, it is enough to consider limit along $x=my$ here.
    $endgroup$
    – Exp ikx
    Feb 1 at 19:20


















$begingroup$
Since the limit should be the same irrespective of the approach chosen, it is enough to consider limit along $x=my$ here.
$endgroup$
– Exp ikx
Feb 1 at 19:20






$begingroup$
Since the limit should be the same irrespective of the approach chosen, it is enough to consider limit along $x=my$ here.
$endgroup$
– Exp ikx
Feb 1 at 19:20












1 Answer
1






active

oldest

votes


















0












$begingroup$

You could also argue like this: Obviously, the partial derivatives are continuous in $mathbb R^2 setminus {(0,0)}$. If they were also continuous in $(0,0)$, the function would be continuously partially differentiable, hence totally differentiable and hence continuous. But $f(1/n,1/n)=1/2$ does not converge to $f(0,0)=0$ for $ntoinfty$.



To be exact, this only shows that $frac{partial f}{partial x}$ or $frac{partial f}{partial y}$ is discontinuous at $(0,0)$, but for symmetry reasons it cannot be only one of them.






share|cite|improve this answer











$endgroup$














    Your Answer





    StackExchange.ifUsing("editor", function () {
    return StackExchange.using("mathjaxEditing", function () {
    StackExchange.MarkdownEditor.creationCallbacks.add(function (editor, postfix) {
    StackExchange.mathjaxEditing.prepareWmdForMathJax(editor, postfix, [["$", "$"], ["\\(","\\)"]]);
    });
    });
    }, "mathjax-editing");

    StackExchange.ready(function() {
    var channelOptions = {
    tags: "".split(" "),
    id: "69"
    };
    initTagRenderer("".split(" "), "".split(" "), channelOptions);

    StackExchange.using("externalEditor", function() {
    // Have to fire editor after snippets, if snippets enabled
    if (StackExchange.settings.snippets.snippetsEnabled) {
    StackExchange.using("snippets", function() {
    createEditor();
    });
    }
    else {
    createEditor();
    }
    });

    function createEditor() {
    StackExchange.prepareEditor({
    heartbeatType: 'answer',
    autoActivateHeartbeat: false,
    convertImagesToLinks: true,
    noModals: true,
    showLowRepImageUploadWarning: true,
    reputationToPostImages: 10,
    bindNavPrevention: true,
    postfix: "",
    imageUploader: {
    brandingHtml: "Powered by u003ca class="icon-imgur-white" href="https://imgur.com/"u003eu003c/au003e",
    contentPolicyHtml: "User contributions licensed under u003ca href="https://creativecommons.org/licenses/by-sa/3.0/"u003ecc by-sa 3.0 with attribution requiredu003c/au003e u003ca href="https://stackoverflow.com/legal/content-policy"u003e(content policy)u003c/au003e",
    allowUrls: true
    },
    noCode: true, onDemand: true,
    discardSelector: ".discard-answer"
    ,immediatelyShowMarkdownHelp:true
    });


    }
    });














    draft saved

    draft discarded


















    StackExchange.ready(
    function () {
    StackExchange.openid.initPostLogin('.new-post-login', 'https%3a%2f%2fmath.stackexchange.com%2fquestions%2f3096620%2fcontinuity-of-partial-derivatives-at-0-0%23new-answer', 'question_page');
    }
    );

    Post as a guest















    Required, but never shown

























    1 Answer
    1






    active

    oldest

    votes








    1 Answer
    1






    active

    oldest

    votes









    active

    oldest

    votes






    active

    oldest

    votes









    0












    $begingroup$

    You could also argue like this: Obviously, the partial derivatives are continuous in $mathbb R^2 setminus {(0,0)}$. If they were also continuous in $(0,0)$, the function would be continuously partially differentiable, hence totally differentiable and hence continuous. But $f(1/n,1/n)=1/2$ does not converge to $f(0,0)=0$ for $ntoinfty$.



    To be exact, this only shows that $frac{partial f}{partial x}$ or $frac{partial f}{partial y}$ is discontinuous at $(0,0)$, but for symmetry reasons it cannot be only one of them.






    share|cite|improve this answer











    $endgroup$


















      0












      $begingroup$

      You could also argue like this: Obviously, the partial derivatives are continuous in $mathbb R^2 setminus {(0,0)}$. If they were also continuous in $(0,0)$, the function would be continuously partially differentiable, hence totally differentiable and hence continuous. But $f(1/n,1/n)=1/2$ does not converge to $f(0,0)=0$ for $ntoinfty$.



      To be exact, this only shows that $frac{partial f}{partial x}$ or $frac{partial f}{partial y}$ is discontinuous at $(0,0)$, but for symmetry reasons it cannot be only one of them.






      share|cite|improve this answer











      $endgroup$
















        0












        0








        0





        $begingroup$

        You could also argue like this: Obviously, the partial derivatives are continuous in $mathbb R^2 setminus {(0,0)}$. If they were also continuous in $(0,0)$, the function would be continuously partially differentiable, hence totally differentiable and hence continuous. But $f(1/n,1/n)=1/2$ does not converge to $f(0,0)=0$ for $ntoinfty$.



        To be exact, this only shows that $frac{partial f}{partial x}$ or $frac{partial f}{partial y}$ is discontinuous at $(0,0)$, but for symmetry reasons it cannot be only one of them.






        share|cite|improve this answer











        $endgroup$



        You could also argue like this: Obviously, the partial derivatives are continuous in $mathbb R^2 setminus {(0,0)}$. If they were also continuous in $(0,0)$, the function would be continuously partially differentiable, hence totally differentiable and hence continuous. But $f(1/n,1/n)=1/2$ does not converge to $f(0,0)=0$ for $ntoinfty$.



        To be exact, this only shows that $frac{partial f}{partial x}$ or $frac{partial f}{partial y}$ is discontinuous at $(0,0)$, but for symmetry reasons it cannot be only one of them.







        share|cite|improve this answer














        share|cite|improve this answer



        share|cite|improve this answer








        edited Feb 2 at 1:46

























        answered Feb 2 at 1:40









        Mars PlasticMars Plastic

        1,455122




        1,455122






























            draft saved

            draft discarded




















































            Thanks for contributing an answer to Mathematics Stack Exchange!


            • Please be sure to answer the question. Provide details and share your research!

            But avoid



            • Asking for help, clarification, or responding to other answers.

            • Making statements based on opinion; back them up with references or personal experience.


            Use MathJax to format equations. MathJax reference.


            To learn more, see our tips on writing great answers.




            draft saved


            draft discarded














            StackExchange.ready(
            function () {
            StackExchange.openid.initPostLogin('.new-post-login', 'https%3a%2f%2fmath.stackexchange.com%2fquestions%2f3096620%2fcontinuity-of-partial-derivatives-at-0-0%23new-answer', 'question_page');
            }
            );

            Post as a guest















            Required, but never shown





















































            Required, but never shown














            Required, but never shown












            Required, but never shown







            Required, but never shown

































            Required, but never shown














            Required, but never shown












            Required, but never shown







            Required, but never shown







            Popular posts from this blog

            Can a sorcerer learn a 5th-level spell early by creating spell slots using the Font of Magic feature?

            ts Property 'filter' does not exist on type '{}'

            mat-slide-toggle shouldn't change it's state when I click cancel in confirmation window